Difference between revisions of "2014 AMC 10A Problems/Problem 11"

(Created page with "==Problem== A customer who intends to purchase an appliance has three coupons, only one of which may be used: Coupon 1: <math>10\%</math> off the listed price if the listed pri...")
 
Line 16: Line 16:
 
\textbf{(D) }\textdollar239.95\qquad
 
\textbf{(D) }\textdollar239.95\qquad
 
\textbf{(E) }\textdollar259.95\qquad</math>
 
\textbf{(E) }\textdollar259.95\qquad</math>
 +
 +
==Solution==
 +
 +
==See Also==
 +
 +
{{AMC10 box|year=2014|ab=A|num-b=10|num-a=12}}
 +
{{MAA Notice}}

Revision as of 23:17, 6 February 2014

Problem

A customer who intends to purchase an appliance has three coupons, only one of which may be used:

Coupon 1: $10\%$ off the listed price if the listed price is at least $\textdollar50$

Coupon 2: $\textdollar 20$ off the listed price if the listed price is at least $\textdollar100$

Coupon 3: $18\%$ off the amount by which the listed price exceeds $\textdollar100$

For which of the following listed prices will coupon $1$ offer a greater price reduction than either coupon $2$ or coupon $3$?

$\textbf{(A) }\textdollar179.95\qquad \textbf{(B) }\textdollar199.95\qquad \textbf{(C) }\textdollar219.95\qquad \textbf{(D) }\textdollar239.95\qquad \textbf{(E) }\textdollar259.95\qquad$

Solution

See Also

2014 AMC 10A (ProblemsAnswer KeyResources)
Preceded by
Problem 10
Followed by
Problem 12
1 2 3 4 5 6 7 8 9 10 11 12 13 14 15 16 17 18 19 20 21 22 23 24 25
All AMC 10 Problems and Solutions

The problems on this page are copyrighted by the Mathematical Association of America's American Mathematics Competitions. AMC logo.png